Sie sind auf Seite 1von 39

GRE AWA John

1: Statistical & Study Errors I.The following appeared in a memo from the vice president of a food distribution company with food storage warehouses in several cities. "Recently, we signed a contract with the Fly-Away Pest Control Company to provide pest control services at our fast-food warehouse in Palm City, but last month we discovered that over $20,000 worth of food there had been destroyed by pest damage. Meanwhile, the Buzzoff Pest Control Company, which we have used for many years, continued to service our warehouse in Wintervale, and last month only $10,000 worth of the food stored there had been destroyed by pest damage. Even though the price charged by Fly-Away is considerably lower, our best means of saving money is to return to Buzzoff for all our pest control services." Write a response in which you discuss what specific evidence is needed to evaluate the argument and explain how the evidence would weaken or strengthen the argument.

II.The following appeared in a memo from the vice president of a food distribution company with food storage warehouses in several cities. "Recently, we signed a contract with the Fly-Away Pest Control Company to provide pest control services at our warehouse in Palm City, but last month we discovered that over $20,000 worth of food there had been destroyed by pest damage. Meanwhile, the Buzzoff Pest Control Company, which we have used for many years in Palm City, continued to service our warehouse in Wintervale, and last month only $10,000 worth of the food stored there had been destroyed by pest damage. Even though the price charged by Fly-Away is considerably lower, our best means of saving money is to return to Buzzoff for all our pest control services." Write a response in which you discuss what questions would need to be answered in order to decide whether the recommendation and the argument on which it is based are reasonable. Be sure to explain how the answers to these questions would help to evaluate the recommendation. However, the vice president does not answer crucial questions which if answered may undermine his recommendation. III.The following appeared in a memo from the vice president of a food distribution company with food storage warehouses in several cities. "Recently, we signed a contract with the Fly-Away Pest Control Company to provide pest control services at our warehouse in Palm City, but last month we discovered that over $20,000 worth of food there had been destroyed by pest damage. Meanwhile, the Buzzoff Pest Control Company, which we have used for many years in Palm City, continued to service our warehouse in Wintervale, and last month only $10,000 worth of the food stored there had been destroyed by pest damage. This difference in pest damage is best explained by the negligence of Fly-Away." Write a response in which you discuss one or more alternative explanations that could rival the proposed explanation and explain how your explanation(s) can plausibly account for the facts presented in the argument. However, the author does not consider other alternative explanations. First, the author assumes that the Analogy: Palm City and Wintervalestored food might be different, climate, proportion of damage to the actual amount of food kept in the warehouse. The amount of money in proportion to its real value. Saving money Hasty decision making: evidence of one month " The Fly-Away Pest-Control Palm , 2 .

GRE AWA John

, Buzzoff Pest-Control , , Wintervale , 1 . Fly-Away , Buzzoff ." : buzzoff pest-control service . 1. bad analogy : . . ( ) 2. \ ( ) 3. . In the memo, the vice president of a food-distribution concludes that in order to save their companys money, they should return to Buzzoff Company of all their pest-control services. To support this argument, the vice president of a food-distribution should provide objective information that proves both cities were in the same environment. The vice president of a food distribution relies on unsubstantiated assumption is therefore unconvincing as it stands. First of all, the vice president of the food distribution provides no information about the weather in two cites. It is quite possible that the fast-food warehouse in Palm City had been destroyed by pest damage because of bad weather condition and the warehouse in Wintervale had been less destroyed by pest damage because of good weather condition. Therefore, without considering the weather condition in two the cites, the vice presidents argument is not reliable. Second of all, the vice president of the food distribution unfairly assumes that both food conditions were exactly the same condition; however, it is possible that the foods condition in warehouse in Palm City was originally worse than the one in Wintervale. Food in bad condition quickly influences other food around it so in this case, it is not all The Fly-Away Pest-Control Companys fault to make over $20,000 worth of food damage. Third of all, the vice president of the food distribution provides no information about how much each company charged for the pest-control service. It is possible that the total price, including the worth of food destroyed by pest damage in Palm City and the pest-control service, is less expensive than paying for Buzzoff Company. In this case, Buzzoff Company charges more money; therefore, the company could not save their money. Fourth of all, it is also possible that Palm City is naturally or geographically vulnerable to pest damage. In this case, it does not matter which company they hire. If they just hire Buzzoff Company for the pest-control service, the result would not be different. Moreover, they would be spending more than hiring The Fly-Away Pest-Control Company. In short, this memo fails to provide the key evidence need to support its claims. To make the claim stronger, the vice president of a food distribution should provide information about weather in two cites and the original food conditions. The vice president of the food distribution also needs to provide information about the price charged by the two companies and natural and geographical information. With providing that information, the vice president of a food distributions argument would be more convincing. Good transitions and organization. Work on smoothening awkward sentence structures. Score: 4.0 Hospital statistics regarding people who go to the emergency room after roller-skating accidents indicate the need for more protective equipment. Within that group of people, 75 percent of those who had accidents in streets or parking lots had not been wearing any protective clothing (helmets, knee pads, etc.) or any light-reflecting material (clip-on lights, glow-in-the-dark wrist pads, etc.). Clearly, the statistics indicate that by investing in high-quality protective gear and reflective equipment, roller skaters will greatly reduce their risk of being severely injured in an accident. Write a response in which you examine the stated and/or unstated assumptions of the argument. Be sure to explain how the argument depends on these assumptions and what the implications are for the argument if the assumptions prove unwarranted.

GRE AWA John

Assumptions: The

author is assuming that 75 percent is a significant number; protective clothing and light reflecting material mitigate severe injuries (the cautious nature of those who wear such equipment might have prevented accidents; or the skills and reflex of the roller skaters; drivers are the main reason for accidents.)

ETS Essay Response Score 5


The argument presented is limited but useful. It indicates a possible relationship between a high percentage of accidents and a lack of protective equipment. The statistics cited compel a further investigation of the usefulness of protective gear in preventing or mitigating roller-skating related injuries. However, the conclusion that protective gear and reflective equipment would "greatly reduce.risk of being severely injured" is premature. Data is lacking with reference to the total population of skaters and the relative levels of experience, skill and physical coordination of that population. It is entirely possible that further research would indicate that most serious injury is averted by the skater's ability to react quickly and skillfully in emergency situations. Another area of investigation necessary before conclusions can be reached is identification of the types of injuries that occur and the various causes of those injuries. The article fails to identify the most prevalent types of roller-skating related injuries. It also fails to correlate the absence of protective gear and reflective equipment to those injuries. For example, if the majority of injuries are skin abrasions and closed-head injuries, then a case can be made for the usefulness of protective clothing mentioned. Likewise, if injuries are caused by collision with vehicles (e.g. bicycles, cars) or pedestrians, then light-reflective equipment might mitigate the occurences. However, if the primary types of injuries are soft-tissue injuries such as torn ligaments and muscles, back injuries and the like, then a greater case could be made for training and experience as preventative measures.

Reader Commentary for Essay Response Score 5


This strong response gets right to the work of critiquing the argument, observing that it "indicates a possible relationship" but that its conclusion "is premature." It raises three central questions that, if answered, might undermine the soundness of the argument: What are the characteristics of the total population of skaters? What is the usefulness of protective or reflective gear in preventing or mitigating roller skatingrelated injuries? What are the types of injuries sustained and their causes? The writer develops each of these questions by considering possible answers that would either strengthen or weaken the argument. The paper does not analyze the argument as insightfully or develop the critique as fully as required for a 6 paper, but the clear organization, strong control of language, and substantial degree of development warrant more than a score of 4. Milk and dairy products are rich in vitamin D and calcium, substances essential for building and maintaining bones. Many people therefore believe that a diet rich in dairy products can help prevent osteoporosis, a disease in which the bones weaken significantly with age and that is linked to both environmental and genetic factors. But a long-term study of a large number of people has found that those who have consistently consumed dairy products throughout the years of the study have a higher rate of bone fractures than any other participants in the study. Since bone fractures are a symptom of osteoporosis, this study result shows that a diet rich in dairy products may actually increase, rather than decrease, the risk of osteoporosis.
Write a response in which you discuss what specific evidence is needed to evaluate the argument and explain how the evidence would weaken or strengthen the argument.

Proven fact: vitamin D and calcium in milk help prevent osteoporosis. Challenge: long-term study of a lot of people: consistently consumed dairy products throughout the years=> higher rate of bone fractures than other participants. Thus, bone fractures(symptom of osteoporosis) increased by diet rich in dairy products.

GRE AWA John

-The author uses unscientific and imprecise terms in his study / to support his claim -Control group -The author tries to refute a well established fact through a single study. -Other factors bring about osteoporosis. For example, occupation hazards, genetics, dietary habits. -Bone fracture could have / might have / may have been caused by other factors: work environment, etc. and not specifically osteoporosis. -The lack of a controlled environment of the study group. The researchers only noted the consumption of milk and did not control other variantslifestyle and diet. -The author bases his conclusion only by noting that the study samples consumed milk. The researchers only noted the consumption of milk and did not control other variantslifestyle and diet. -Dairy products might have had adverse effects on some of the participants in the study, for example, lactose intolerance. In this study, the author claims that the consumption of milk actually increases the risk of osteoporosis. To support this claim, the author refers to a long-term study he conducted. Thesis However, careful scrutiny of this study reveals that it provides little credible support for the authors claim First, the author states that the bone fractures were caused by osteoporosis. However, there could be other reasons for the bone fractures. For example, it is entirely possible that working conditions might have contributed to the bone fracture. A person working at a construction has a much higher chance of bone fracture than a person that works in an office. The author needs to provide the reasons for the occurrence of the study participants bone fractures. Thus, the author should provide more sufficient evidence that .. The author observes a correlation between osteoporosis and bone fracture, then concludes that the former is the cause of the latter. However, the author fails to rule out other possible explanations. For example, it is entirely possible that working The authors study includes an indefinite amount of people whose backgrounds arent defined: genetic(family) history and environment; the age at which the participants partook in the study. It may be that the participants had genetic predisposition that were more prone to osteoporosis. The author does not sufficiently define what types of people were included in the study. Bones are affected as noted in the prompt by various factors: genes and environment. The author, however, does not Finally, the author asserts his claim definitively. While it is a proven fact that dairy products help people maintain healthy bones, the author overthrows this long standing fact with a single long-term study that he did. D . , , , . , , . , , . : . 1. . ( , ..) . , . . 2. . .( ) 3. . . 4. . . ( )

A recently issued twenty-year study on headaches suffered by the residents of Mentia investigated the possible therapeutic effect of consuming salicylates. Salicylates are members of the same chemical

GRE AWA John

family as aspirin, a medicine used to treat headaches. Although many foods are naturally rich in salicylates, food-processing companies also add salicylates to foods as preservatives. The twenty-year study found a correlation between the rise in the commercial use of salicylates and a steady decline in the average number of headaches reported by study participants. At the time when the study concluded, food-processing companies had just discovered that salicylates can also be used as flavor additives for foods, and, as a result, many companies plan to do so. Based on these study results, some health experts predict that residents of Mentia will suffer even fewer headaches in the future. Write a response in which you discuss what questions would need to be answered in order to decide whether the prediction and the argument on which it is based are reasonable. Be sure to explain how the answers to these questions would help to evaluate the prediction. In this study, the author asserts that the number of headaches suffered by the average citizen of Mentia will steadily decline though the use of salicylates as flavor additives. To support this assertion, the author provides several evidences to support his claim. However, his argument is specious. Unscientific data: correlation, the specific amount used as preservatives, Other factors could have caused the decline in headaches. 20 year study: Was it a controlled study? Lack of controlled environment exposes the participants to other factors. 1) The decline in the average number of headaches does not necessarily substantiate that it was indeed caused by the commercial use of salicylates. It is entirely possible that other factors are responsible for the decline. Perhaps, other substances in the food remedied the headaches of the participants. Or the decline in headache may have been caused by other dietary consumption or lifestyle habits like exercising. Accordingly, if these are true, the author cannot draw any firm conclusion that the commercial use of salicylates caused the decline in the average number of headaches. " , . , . . , . , Mentia ." : . 1. Salicylate headache . -> : . 2. Salicylate headache . -> : headache . 3. headache Salicylate . -> : . 4. Flavor additive Salicylate headache . -> : Flavor additive . ? The following appeared as part of an article in a business magazine. "A recent study rating 300 male and female Mentian advertising executives according to the average number of hours they sleep per night showed an association between the amount of sleep the executives need and the success of their firms. Of the advertising firms studied, those whose executives reported needing no more than 6 hours of sleep per night had higher profit margins and faster growth. These results suggest that if a business wants to prosper, it should hire only people who need less than 6 hours of sleep per night." Write a response in which you examine the stated and/or unstated assumptions of the argument. Be sure to explain how the argument depends on these assumptions and what the implications are for the

GRE AWA John

argument if the assumptions prove unwarranted.


Assumptions: 1. Correlation of hours of sleep with success. The author assumes that the executives used their wake-up hours on work? Could there be other factors? 2. Executives versus employeescriteria for hiring is too simpleminded. 3. Characteristics of the 300 male and female executives. 4. The study was about advertising executives but the author applies this case to all businesses. 5. The author should consider long-term stability rather that sharp growth and profit. 6. Average number of hours of sleep is insufficient to support the authors recommendation. 7. The author assumes that this lifestyle pattern will remain consistent.

The following appeared in a memo from the president of a company that makes breakfast cereals. In a recent study, subjects who ate soybeans at least five times per week had significantly lower cholesterol levels than subjects who ate no soy products. By fortifying our Wheat-O cereal with soy protein, we can increase sales by appealing to additional consumers who are concerned about their health. This new version of Wheat-O should increase company profits and, at the same time, improve the health of our customers. Write a response in which you examine the stated and/or unstated assumptions of the argument. Be sure to explain how the argument depends on these assumptions and what the implications are for the argument if the assumptions prove unwarranted. " , 5 . Wheat-O , . , ." : By increasing our Wheat-O cereal with soy protein, we can multiply sales 1. . . 2. . 3. . 4. .( .)

GRE AWA John

In a study of the reading habits of Waymarsh citizens conducted by the University of Waymarsh, most respondents said that they preferred literary classics as reading material. However, a second study conducted by the same researchers found that the type of book most frequently checked out of each of the public libraries in Waymarsh was the mystery novel. Therefore, it can be concluded that the respondents in the first study had misrepresented their reading habits. Write a response in which you discuss what specific evidence is needed to evaluate the argument and explain how the evidence would weaken or strengthen the argument.

Evidence needed: -Information about the respondentsage, gender, class, precise number of respondents. -Time-shift. After how many years was the second study conductedtaste in books may change over time. -Can library records sufficiently evidence the reading habits of Waymarsh citizens.
1) 2) 3) 4) 5)

6) First, the author provides no evidence that the surveys studys results are statistically reliable. Lacking

information about the precise methodology of the study, the number of (customers)respondents surveyed and the number of respondentsvarious information such as gender, age, and social background, which are essential to bolster the conclusion, it is impossible to assess the validity of the results. It is possible that people who feel inclined to( take low fat and low cholesterol foods ) read literary classics were more willing to respond to the survey than were others. Another problem is the representativeness of the respondents. Were they representative of all the customersWaymarsh citizens? Were they chosen for the survey randomly? Lastly, the survey results must depend on the honesty and integrity of the respondents. Without more information about the survey, the author cannot simply conclude that (most of customers want to take low fat and low cholesterol foods)Waymarsh citizens misrepresented their reading habit on the first study on the basis of this surveystudy.(survey reliability)
The exact number of books checked-out should be noted. Mystery novels could have been in trend at the time of the study. Public libraries vs. other libraries such as Leeville University library? Leeville Leeville , . , , Leeville . , . : , it can be concluded that the respondents in the first study had misrepresented their reading habits 1. . 2. .

High Frequency Group 2: National->Local, Local->National & Group Member Error

GRE AWA John

The following appeared in a letter to the editor of a Batavia newspaper. "The department of agriculture in Batavia reports that the number of dairy farms throughout the country is now 25 percent greater than it was 10 years ago. During this same time period, however, the price of milk at the local Excello Food Market has increased from $1.50 to over $3.00 per gallon. To prevent farmers from continuing to receive excessive profits on an apparently increased supply of milk, the Batavia government should begin to regulate retail milk prices. Such regulation is necessary to ensure fair prices for consumers." Write a response in which you discuss what questions would need to be answered in order to decide whether the recommendation is likely to have the predicted result. Be sure to explain how the answers to these questions would help to evaluate the recommendation. First, the author points to the fact that 25% increase of dairy farms may be(could be, might be) due to population increase. The milk price increase could be reflecting the increase in cost of living. The actual price of milk might be cheaper if the economy is experiencing inflation. One market, Excello Food, cannot reflect the price increase of every market in Batavia. The author claims that farmers are receiving excessive profits; however, this might not be the case. The author claims that by regulating milk prices, the government will successfully ensure both lower prices and an adequate supply of milk for consumers. However, this might not be the best solution.
The writer adduces the information released by the department of agriculture that the number of dairy farms has increased by 25 percent over the last 10 years and thus he claims that dairy farmers are earning excessive profits from milk. However, the information is filled with loops and holes which need to be filled. First, what percentage of the milk are sold as milk? Are the dairy farms processing the milk into other dairy products? Or are they selling them as milk?What percentage of the milk is sold as milk? Could this be a natural increase? Can the price at the local Excello Food Market represent the price of milk in stores throughout all of Batavia? Are the farmers receiving excessive profits? What is the cost of producing milk? Could such regulation offer fair prices to consumers?

Topic Sentence: To begin with, the author assumes that the price of Excello Food Market, which has increased from $1.50 to over $3.00 per gallon can be applied to the whole nation of Batavia; however, the author fails to provide evidence to substantiate this assumption. Example First, In addition. Therefore, in order to make the argument more reliable, the author should consider the prices of a large number markets all over the country. There could be an economic inflation. Batavia , 10 25% . , , Excello Food Market 1.5 3 . , . ." : the Batavia government should begin to regulate retail milk prices : ( Excello Food Market doesnt reflect the whole of Batavia) inflation . .( )

GRE AWA John

.( ) .

A recent sales study indicates that consumption of seafood dishes in Bay City restaurants has increased by 30 percent during the past five years. Yet there are no currently operating city restaurants whose specialty is seafood. Moreover, the majority of families in Bay City are two-income families, and a nationwide study has shown that such families eat significantly fewer home-cooked meals than they did a decade ago but at the same time express more concern about healthful eating. Therefore, the new Captain Seafood restaurant that specializes in seafood should be quite popular and profitable. Write a response in which you discuss what questions would need to be addressed in order to decide whether the conclusion and the argument on which it is based are reasonable. Be sure to explain how the answers to the questions would help to evaluate the conclusion.
BAY CITY. The name implies that this is a port city which would have traditionally consumed seafood. 30%--what type of seafood? Unprocessed or processed cans of tuna and sardines? If the latter increased Captain Seafood may not profit. 30%--natural inflation? Are the non-seafood restaurants selling seafood? If yes, this may suffice since patrons would not easily dine at an unfamiliar restaurant. Nationwide studys representativeness. Desire does not lead to action.

Nationwide study applied to Bay City: Eating fewer home cooked meals than they did a decade ago Healthy food. The author needs to show that 30% -> population increase Other foods may be more popular Current number of restaurants that serve seafood might suffice What kind of healthy food are they interested in? Surely not just seafood! The locals may still patronize the current restaurants that serve seafood dishes. < national-> local / local->national > 1) The author assumes that the nationwide trend reflects the general trend upon which the argument relies. Yet, the author fails to provide evidence to substantiate this crucial assumption. The nationwide trend of two income families eating fewer home-cooked and concern for health my not reflect the lifestyle and interests of Bay City citizens. Therefore, to make his argument stronger, the author needs to provide evidence that two income families of Bay City maintains the nationwide trend. , Bay 5 30% . , . , , 10 , . , , . 1. . 2. . . 3. . Bay .

GRE AWA John

Scientists studying historical weather patterns have discovered that in the mid-sixth century, Earth suddenly became significantly cooler. Although few historical records survive from that time, some accounts found both in Asia and Europe mention a dimming of the sun and extremely cold temperatures. Either a huge volcanic eruption or a large meteorite colliding with Earth could have created a large dust cloud throughout Earth's atmosphere that would have been capable of blocking enough sunlight to lower global temperatures significantly. A large meteorite collision, however, would probably create a sudden bright flash of light, and no extant historical records of the time mention such a flash. Some surviving Asian historical records of the time, however, mention a loud boom that would be consistent with a volcanic eruption. Therefore, the cooling was probably caused by a volcanic eruption. Write a response in which you discuss what questions would need to be addressed in order to decide whether the conclusion and the argument on which it is based are reasonable. Be sure to explain how the answers to the questions would help to evaluate the conclusion.

Historical records may not be enough to explain the global cooling phenomenon. There may be reasons other than the two hypothesis to explain the global cooling. A volcanic eruption big enough to produce dust clouds that envelop the earth to cause global cooling would have countless records. The author argues that a large meteorite collision is not a feasible explanation for the global cooling because there are no historical records that indicate a flash. The authors logic is flawed in eliminating the meteorite hypothesis by claiming that there was no historical record of a bright flash of light. According to common sense, a meteorite collision big enough to produce dust that could envelop the earth would result in catastrophe even to the extinction of major species of animals. Loud boom would be insufficient evidence to conclude it was a volcanic eruption. If could hear a loud boom, there should be records of a volcanic eruption because the author claims that the dust cloud from this gigantic eruption was significant enough to cause global cooling. Cooling could have been caused by factors besides volcanic eruption and meteor collision. The absence of historical records that indicate a bright flash of light. Collision could have occurred at a place where there no witnesses. Could have fell out in the sea. Boom might have been caused by things other than a volcanic eruption. Historical records from Asia and Europe may not be applicable to earth as a whole. 6 . , . . , , . , . , . : the cooling was probably caused by a volcanic eruption. : ( .) . (, ) .( , . ) .( , .) . . .

GRE AWA John

The following appeared in a memo from the president of Bower Builders, a company that constructs new homes. "A nationwide survey reveals that the two most-desired home features are a large family room and a large, well-appointed kitchen. A number of homes in our area built by our competitor Domus Construction have such features and have sold much faster and at significantly higher prices than the national average. To boost sales and profits, we should increase the size of the family rooms and kitchens in all the homes we build and should make state-of-the-art kitchens a standard feature. Moreover, our larger family rooms and kitchens can come at the expense of the dining room, since many of our recent buyers say they do not need a separate dining room for family meals." Write a response in which you examine the stated and/or unstated assumptions of the argument. Be sure to explain how the argument depends on these assumptions and what the implications are for the argument if the assumptions prove unwarranted.
The presidents first unstated but apparent assumption is that a nationwide survey can be applied to local areas. There is no guarantee that the area in which Bower Builders builds houses will follow the national consumer trend. Since he only mentions the overall result of the nationwide survey, it is difficult to determine more specific conditions, such whether people in urban and rural areas have the same housing preferences. Such uncertainty is further exacerbated by the nature of the survey question itself it does not deal with current trends in actual home purchases, but with desired home features. The nave assumption that consumer desire will directly result in consumer action underlies the presidents interpretation and application of the survey results to his company plan. Not everyone who wishes for a large family room and kitchen will or can actually buy a house with those features, mainly due to financial reasons. Furthermore, the president also assumes for no evident reason that consumers will not hesitate to purchase houses with state-of-the-art kitchens as a standard rather than optional feature, when it is clear that such a feature will raise the overall cost significantly. He continues to blunder in his mistaken assumptions about consumer behavior in assuming that the taste of recent buyers can represent the concern of future prospective buyers. The fact that recent buyers have claimed no need for separate dining rooms does not mean future buyers will feel the same way as well.

The president of Bower Builders recommends that in order to raise company profits, the company should build houses with large family rooms and large state-of-the-art kitchens. He uses the results of a nationwide survey on desired home features and the example of Bower Builders competitor, Domus Construction, to support his plan. His memo manifests several assumptions about surveys, consumer behavior and Domus Constructions houses that do not necessarily bolster/buttress his argument.

The writer assumes: 1. the nationwide survey results can be applied to the local area (Desire cannot reflect consumer trend.) 2. Domus Construction was profitable because they had such features (The writer should investigate what other features the DC provides and the actual number of homes sold) 3. State-of-the-art kitchens will sell well (no evidence to buttress his assumption; furthermore, they would need to spend more money which would increase the price of the housestheir market will be limited to the affluent) 4. The recent buyers represent the concern of most buyersthat they are o.k. with a house that is without a dining room.

Nationwide survey might not be applicable to this region. Domus Construction could have other superior qualities than Bower Builders. Ex. Design Selling fast doesnt indicate quantity. Small yards: complaints could be voiced in the future.

GRE AWA John

First, the author concludes that a nationwide survey reveals that the two most desired home features are a bathroom with a whirlpool tub and a large kitchen. However, this nationwide survey may not reflect the trends of the customers that Bower Builders target. The author assumes that the nationwide trend reflects customer trends. The national study would lead support only if the nationwide trend reflect the home-purchasing trends. The nationwide trend may just be a trend of desire which does not result in the act of purchasing. However, the author does not provide credible evidence that this is the case. " , 2 . Domus Construction , . , 2 . , ." : We should include whirlpool tubs and a large kitchen. 1. Domus Construction (construction material) . 2. . 3. editor . . 4. .

2011. 7. 19 , GRE AWA OO Argument 94 The president of Bower Builders contends recommends that in order to raise company profits, the company should build houses with large family rooms and large state-of-the-art kitchens. He uses the results of a nationwide survey on desired home features and the example of Bower Builders competitor, Domus Construction, to support his plan. His memo manifests several assumptions about surveys, consumer behavior and Domus Constructions houses that do not necessarily bolster/buttress his argument. The presidents first unstated but apparent assumption is that a nationwide survey can be applied to local areas. There is no guarantee that the area in which Bower Builders builds houses will follow the national consumer trend. Since he only mentions the overall result of the nationwide survey, it is difficult to determine more specific conditions, such whether people in urban and rural areas have the same housing preferences. Such uncertainty is further exacerbated by the nature of the survey question itself it does not deal with current trends in actual home purchases, but with desired home features. The nave assumption that consumer desire will directly result in consumer action underlies the presidents interpretation and application of the survey results to his company plan. Not everyone who wishes for a large family room and kitchen will or can actually buy a house with those features, mainly due to financial reasons. Furthermore, the president also assumes for no evident reason that consumers will not hesitate to purchase houses with state-of-the-art kitchens as a standard rather than optional feature, when it is clear that such a feature will raise the overall cost significantly. He continues to blunder in his mistaken assumptions about consumer behavior in assuming that the taste of recent buyers can represent the concern of future prospective buyers. The fact that recent buyers have claimed no need for separate dining rooms does not mean future buyers will feel the same way as well. In addition, the president finds a real-life actualization of the nationwide survey results in the recent sales of Domus Construction. However, he easily assumes that large family rooms and kitchens are the only reasons the houses of Domus Construction sell well. He does not take into

GRE AWA John

account other features and selling points of the competitors houses. There may well be other explanations for its houses selling more quickly and expensively, including additional home features as well as external factors such as proximity to better schools or superior financial solvency of its clientele. If Bower Builders merely added larger family rooms and kitchens without taking into consideration the other factors, they may lose rather than gain profits. Clearly, the presidents assertion that Bower Builders make houses with large family rooms and high-tech kitchens at the expense of dining rooms rests on a number of assumptions that are ill-informed and nave. If Bower Builders undertakes the proposed plan without further research into local consumer desires, purchasing trends, and the marketing and sales of competing companies, the company will risk losing money by building big new houses that people cannot afford to, or will not wish to buy.

The following appeared in a letter to the editor of a journal on environmental issues. "Over the past year, the Crust Copper Company (CCC) has purchased over 10,000 square miles of land in the tropical nation of West Fredonia. Mining copper on this land will inevitably result in pollution and, since West Fredonia is the home of several endangered animal species, in environmental disaster. But such disasters can be prevented if consumers simply refuse to purchase products that are made with CCC's copper unless the company abandons its mining plans." Write a response in which you examine the stated and/or unstated assumptions of the argument. Be sure to explain how the argument depends on these assumptions and what the implications are for the argument if the assumptions prove unwarranted.

The writer assumes: 1. The writer is trying to avoid an inevitability 2. Mining copper will result pollution (they could make preventative measures.) 3. The writer may be too late from stopping CCC from developing the area into a copper mine 4 Since mining is an underground enterprise, the surface may not be affected that much; therefore, endangered species may not be affected 4. Consumers wont buy CCC products if the journal publishes a negative review about CCC (How many readers?) CCC could a company that has ties with many IT companies and industries in that their copper is almost ubiquitous in various products.

" , CCC(Consolidated Copper ) 1 . , , . , , CCC ." : such disaster can be prevented if consumers simply refuse to purchase products that are made with CCC's copper until the company abandons its mining plans 1. .( ) 2. . 3. . ( .) 4. . 5. ( .)

GRE AWA John

The following is a letter to the editor of an environmental magazine. "In 1975 a wildlife census found that there were seven species of amphibians in Xanadu National Park, with abundant numbers of each species. However, in 2002 only four species of amphibians were observed in the park, and the numbers of each species were drastically reduced. There has been a substantial decline in the numbers of amphibians worldwide, and global pollution of water and air is clearly implicated. The decline of amphibians in Xanadu National Park, however, almost certainly has a different cause: in 1975, troutwhich are known to eat amphibian eggswere introduced into the park." Write a response in which you discuss what specific evidence is needed to evaluate the argument and explain how the evidence would weaken or strengthen the argument.

Evidence needed: the identity of the sender and the census taker of 75 and 02a scientist or an environmentalist.. credibility issue, census methodology vs. mere observationthe absence of evidence is not an evidence of absence, the season of when the census was taken; time-shiftconditions may have changed, worldwide decline may include Xanadu; other species of predators that prey on amphibians because trout is only one species that prey on amphibians; the number of trouthave they increased significantly since 75. First, the author needs to be more overt about the credibility of the census and observation. There were to accounts that notes the population of amphibiansthe first a census and the second an observation. The author needs to bolster his conclusion with the evidence that indicate that the census and second observation were done using scientific methodologies. This could either could strengthen or weaken his claim. In addition, he needs to provide the specific season of when the census and observation occurred. In this letter, the writer is informing an editor of an environmental magazine that the number of amphibians was greatly reduced since 1975, and he points out the introduction of trout as the only reason for the decline. However, the author fails to provide crucial evidences that could strengthen or weaken his conclusion.

The following appeared in a memorandum from the president of Hyper-Go Toy Company. "Last year, sales of our Fierce Fighter toy airplane declined sharply, even though the toy had been a top seller for three years. Our customer surveys show that parents are now more worried about youthful violence and are concerned about better education for their children. Therefore, to maintain profits we should discontinue all our action toys and focus exclusively on a new line of educational toys. Several other toy companies have already begun marketing educational toys and report sales increases last year of 200 percent. And since the average family income is growing, sales of new Hyper-Go toys should also increase." Write a response in which you discuss what specific evidence is needed to evaluate the argument and explain how the evidence would weaken or strengthen the argument.

GRE AWA John

First, the president indicates that the sales of Fierce Fighter toy airplane declined sharply. However, he fails to consider the fact that toys are a fad. Since FFT enjoyed a three year success, it may be natural that the trend would subside. Second, Customer survey: is this representative of most toy consumers? This trend is not new. Third, other companies may have profited not through educational but other toys. Or educational toy profit may be small in proportion to their sale of other toys. 200% Parents are not the customers, companies should concentrate on what the children want to increase profit. Lastly, parents worry about youthful violence and concern for better education are not new trends. The presidents decision to discontinue all action toys and focus only on educational toys is too extreme. If this decision is taken into effect, its procedure may be an onerous task because it would require major shifts in human resources and company image. Therefore, further data should be considered before following up on this decision.
" 3 Fierce Fighter . . , , . 200% . , Hyper-Go ."

3: Time-shift Error Woven baskets characterized by a particular distinctive pattern have previously been found only in the immediate vicinity of the prehistoric village of Palea and therefore were believed to have been unique to the Palean people. Recently, however, archaeologists discovered such a "Palean" basket in Lithos, an ancient village across the Brim River from Palea. The Brim River is very deep and broad, and so the ancient Paleans could only have crossed it by boat, but there is no evidence that the Paleans had boats. And boats capable of carrying groups of people and cargo were not developed until thousands of years after the Palean people disappeared. Moreover, Paleans would have had no need to cross the riverthe woods around Palea are full of nuts, berries, and small game. It follows that the so-called Palean baskets were not unique to Palea. Write a response in which you discuss what specific evidence is needed to evaluate the argument and explain how the evidence would weaken or strengthen the argument. Refutation Prehistoric: time shift->brim river could have been narrow and shallow or it might have not existed. Indigenous patterns may exist in other disconnected remote places. Nuts, berries, small game(hunting) may not have existed or the author should be proved these existed at that time. Abundance of resources doesnt support the reason for seclusion. Boats: not yet found, baskets may have been carried across by the river current without the help of a boat. Lithos might have crossed the river for commercial purpose. One Palean basket does not substantiate the authors claim. The absence of evidence is not an evidence of absence. The author assumes without justification that present conditions are the same as at the prehistoric era. The author unfairly infers from the presence of Brim River which exist today that it would have existed in the past. However, the author fails to offer any evidence to substantiate this inference. It is very likely that the Brim River might not have existed in prehistoric times or if it did exist, may have been shallow and narrow enough for the Paleans to easily cross. Any of these scenarios, if true, would serve to undermine the claim that..

GRE AWA John

First, the author claims that the Brim River was very deep and broad so the Paleans could not have crossed it. However, the author fails to offer any evidence to substantiate that this was true in the prehistoric time of the Paleans. For all we know, the Brim River might not have existed in the prehistoric era or if it did exist, could have been a shallow and narrow river. For example, scientists believe that thousands of years ago an ice-bridge existed on the Bering Sea connecting Eurasia to what is now North America to explain how the Eskimos and the Asian inhabitants of America came to migrate all over the Americas. In this example, the absence of evidence is not an evidence of absence. Therefore, to sufficiently support his claim, the author needs to substantiate the fact that the Brim River really did exist and was broad and wide in the Prehistoric Era, proving that the Paleans could not have influenced or traded with other groups of people.. First, the author claims that the Brim River was very deep and broad so the Paleans could not have crossed it. However, the author fails to offer any evidence to substantiate that this was true in the prehistoric times of the Paleans. For all we know, the Brim River might not have existed in the prehistoric era or if it did exist, could have been a shallow and narrow river. For example, rivers are created by natural erosion over thousands of years. Niagara Falls carved its way from the mouth of Lake Ottawa and created a long river. Likewise, the current Brim Rivers physical features may not have been wide and deep. Therefore, to sufficiently support his claim, the author needs to substantiate the fact that the Brim River really did exist and was broad and wide in the Prehistoric Era, proving that the Paleans could not have influenced or traded with other groups of people..

Palea , Palea . , Lithos "Palean" , Brim Palea . , Palea . , . , . , Palea , , , . , Palean Palea ." : if follows that the so-called Palean baskets were not unique to Palea causal. Bad analogy . . ( ) , . . (..)

Thirteen years ago, researchers studied a group of 25 infants who showed signs of mild distress when exposed to unfamiliar stimuli such as an unusual odor or a tape recording of an unknown voice. They discovered that these infants were more likely than other infants to have been conceived in early autumn, a time when their mothers' production of melatonin hormone known to affect some brain functions would naturally increase in response to decreased daylight. In a follow-up study conducted earlier this year, more than half of these children now teenagers who had shown signs of distress identified themselves as shy. Clearly, increased levels of melatonin before birth cause shyness during infancy and this shyness continues into later life. Write a response in which you examine the stated and/or unstated assumptions of the argument. Be sure to explain how the argument depends on these assumptions and what the implications are for the argument if the assumptions prove unwarranted.

GRE AWA John

Any baby exposed to unpleasant stimuli would react in such way. First of all, the author states 25 infants as his evidence. However, this research sample is too small to prove his claim. Second, the author states that 25 infants were conceived in early autumn, which he claims lead to a shy disposition. However, this is faulty evidence. Third, the research study was a long term study done in the span of 13 years. However, the author only writes about the initial and final stages of the study and leaves out evidences of what could have happened during the 13 years which could be more evidential factors of influence. Fourth, neither the infants genetic predisposition nor their environment were taken into account. Finally, the author concludes that his shyness continues into later life. (Other factors could alter this disposition: epigenetic theory,) 13 25 , . , , . , 10 . , , , . : Clearly, increased levels of melatonin before birth cause shyness during infancy and this shyness continues into later life. 1. 25 . 2. 3. .( , )

The following is a letter to the editor of the Atticus City newspaper. "Former Mayor Durant owes an apology to the city of Atticus. Both the damage to the River Bridge, which connects Atticus to Hartley, and the traffic problems we have long experienced on the bridge were actually caused 20 years ago by Durant. After all, he is the one who approved the construction of the bridge. If he had approved a wider and better-designed bridge, on which approximately the same amount of public money would have been spent, none of the damage or problems would have occurred. Instead, the River Bridge has deteriorated far more rapidly over the past 20 years than has the much longer Derby Bridge up the river. Even though the winters have been severe in the past several years, this is no excuse for the negligence and wastefulness of Durant." Write a response in which you discuss what questions would need to be answered in order to decide whether the recommendation is likely to have the predicted result. Be sure to explain how the answers to these questions would help to evaluate the recommendation.

" Durant Atticus . Atticus Hartley River Bridge

GRE AWA John

20 . . , . 20 Derby . , ." : Former Mayor Durant owes an apology to the city of Atticus 1. 2. 3. 4. . ( .) . . .

4: (False Cause)
Fifteen years ago, Omega University implemented a new procedure that encouraged students to evaluate the teaching effectiveness of all their professors. Since that time, Omega professors have begun to assign higher grades in their classes, and overall student grade averages at Omega have risen by 30 percent. Potential employers, looking at this dramatic rise in grades, believe that grades at Omega are inflated and do not accurately reflect student achievement; as a result, Omega graduates have not been as successful at getting jobs as have graduates from nearby Alpha University. To enable its graduates to secure better jobs, Omega University should terminate student evaluation of professors. Write a response in which you discuss what specific evidence is needed to evaluate the argument and explain how the evidence would weaken or strengthen the argument.

Omega: professor evaluation implemented 15 years ago => Omega prof. assign higher grades 30% Employers believe theres grade inflation Thus, unsuccessful employment than Alpha Therefore: to secure jobs Omega should end evaluating profs.

GRE AWA John

Specific evidence needed: Relationship between higher grades and evaluation Relationship between GPA and unsuccessful employment Alphas education could just be better than Omega Fifteen years ago is a long time other factors could have influenced. Why is the inflation a problem just now? How much is Alpha better. Is the comparison just? How much gap is there. Omegas student could just be doing better in their studies. Comparison to other universities? Could Omega alleviate the employment problem by implementing a different procedure or program? "15 , . , , 30% . . , Alpha . ." : Omega University should now terminate student evaluation of professors. : ( ) . . Alpha . Alpha .

In this memo, the dean of Omega University(OU) recommends OU to terminate professor evaluation to secure better jobs for the students. To support this recommendation, the dean offers several reasons. However, this argument contains several logical flaws which render it unconvincing. A threshold problem with the argument involves the voluntary nature of the evaluation procedure. The dean provides no evidence about the number or percentage of Omega students who participate in the procedure. Lacking such evidence it is entirely possible that those numbers are insignificant, in which case terminating the procedure is unlikely to have any effect on the grade average of Omega students or their success in getting jobs after graduation. The argument also assumes unfairly that the grade-average increase is the result of the evaluation procedure--rather than some other phenomenon. The dean ignores a host of other possible explanations for the increase--such as a trend at Omega toward higher admission standards, or higher quality instruction or facilities. Without ruling out all other possible explanations for the grade-average increase, the dean cannot convince me that by terminating the evaluation procedure Omega would curb its perceived grade inflation let alone help its graduates get jobs. Even if the evaluation procedure has resulted in grade inflation at Omega, the dean's claim that grade inflation explains why Omega graduates are less successful than Alpha graduates in getting jobs is unjustified. The dean overlooks a myriad of other possible reasons for Omega's comparatively poor job-placement record. Perhaps Omega's career services are

GRE AWA John

inadequate; or perhaps Omega's curriculum does not prepare students for the job market as effectively as Alpha's. In short, without accounting for other factors that might contribute to Omega graduates' comparative lack of success in getting jobs, the dean cannot justify the claim that if Omega curbs its grade inflation employers will be more likely to hire Omega graduates. Finally, even if the dean can substantiate all of the foregoing assumptions, the dean's assertion that Omega must terminate its evaluation procedure to enable its graduates to find better jobs is still unwarranted, in two respects. First, the dean ignores other possible ways by which Omega can increase its job-placement record--for example, by improving its public relations or career-counseling services. Second, the dean unfairly equates "more" jobs with "better" jobs. In other words, even if more Omega graduates are able to find jobs as a result of the dean's recommended course of action, the kinds of jobs Omega graduates find would not necessarily be better ones. In sum, the dean's argument is unpersuasive as it stands. To strengthen it the dean must provide better evidence that the increase in grade average is attributable to Omega's professor-evaluation procedure, and that the end result is a perception on the part of employers that Omega graduates are less qualified for jobs than Alpha graduates. To better assess the argument I would need to analyze 15-year trends in (l) the percentage of Omega students participating in the evaluation procedure, (2) Omega's admission standards and quality of education, and (3) Omega's emphasis on job training and career preparation. I would also need to know what other means are available to Omega for enabling its graduates to find better jobs.

The following appeared in a memo from a vice president of Quiot Manufacturing. "During the past year, Quiot Manufacturing had 30 percent more on-the-job accidents than at the nearby Panoply Industries plant, where the work shifts are one hour shorter than ours. Experts say that significant contributing factors in many on-the-job accidents are fatigue and sleep deprivation among workers. Therefore, to reduce the number of on-the-job accidents at Quiot and thereby increase productivity, we should shorten each of our three work shifts by one hour so that employees will get adequate amounts of sleep." Write a response in which you examine the stated and/or unstated assumptions of the argument. Be sure to explain how the argument depends on these assumptions and what the implications are for the argument if the assumptions prove unwarranted. The following appeared in a memo from a vice president of Alta Manufacturing. "During the past year, Alta Manufacturing had thirty percent more on-the-job accidents than nearby Panoply Industries, where the work shifts are one hour shorter than ours. Experts believe that a significant contributing factor in many accidents is fatigue caused by sleep deprivation among workers. Therefore, to reduce the number of on-the-job accidents at Alta, we recommend shortening each of our three work shifts by one hour. If we do this, our employees will get adequate amounts of sleep." Write a response in which you discuss what questions would need to be answered in order to decide whether the recommendation and the argument on which it is based are reasonable. Be sure to explain how the answers to these questions would help to evaluate the recommendation.

GRE AWA John

The following appeared in a memo from the vice president of Butler Manufacturing. "During the past year, workers at Butler Manufacturing reported 30 percent more on-the-job accidents than workers at nearby Panoply Industries, where the work shifts are one hour shorter than ours. A recent government study reports that fatigue and sleep deprivation among workers are significant contributing factors in many on-the-job accidents. If we shorten each of our work shifts by one hour, we can improve Butler Manufacturing's safety record by ensuring that our employees are adequately rested." 1. 2. Write a response in which you discuss what specific evidence is needed to evaluate the argument and explain how the evidence would weaken or strengthen the argument. Write a response in which you discuss what questions would need to be answered in order to decide whether the recommendation is likely to have the predicted result. Be sure to explain how the answers to these questions would help to evaluate the recommendation.

4 Alta has 30% more job accidents than Panoply(work shifts one hour shorter) Experts: Job accidents caused by fatigue and sleep deprivation. Therefore, to reduce job accidents and increase productivity, shorten three work shifts by one hour for adequate sleep. ", Panoply Industries 30% . 1 . . , , , 1 3 ." In this memo, the (author) vice president of Alta Manufacturing (AM) recommends that to reduce on-the-job accidents and increase productivity, AM should shorten its three work shifts by one hour so that employees can get more sleep. To support this recommendation, the author provides several evidences. However, careful scrutiny of each of the facts reveals that it provides little credible support for the authors recommendation. Questions: The number of accidents? What kind of accidents? The seriousness of the accidents is important. How many employees are in each company? What are their products? False cause: Sleep may not be the reason for the on-the-job accidents. What do Alta and Panoply manufacture? First of all, the author believes that fatigue caused the on-the job accidents. However, there could be other reasons. The author observes a correlation between sleep deprivation and on-the-job accidents then concludes that the former is the cause of the latter. However, the author fails to rule out other possible explanations. For example, it is entirely possible that Alta factories require more strenuous and dangerous labor than Panoply. Without ruling out all other such factors it is unfair to conclude that fatigue is responsible for the accidents. In addition, the work-shifts may not be the cause of the sleep deprivation and fatigue. It is possible Thus, the author should provide what exactly Panoply and Alta manufacture, and more precise data about their working conditions to be more convincing. Shortening the shift by one hour does not necessarily lead to more sleep. And is one hour enough. Less accidents does not mean increased productivity. : We should shorten each of out three work shifts by one hour : . , , . . .( ) .( .)

GRE AWA John

This editorial recommends that Alta Manufacturing reduce its work shifts by one hour each in order to reduce its on-the-job accident rate and thereby increase Alta's productivity. To support this recommendation the author points out that last year the number of accidents at Alta was 30% greater than at Panoply Industries, where work shifts were one hour shorter. The author also cites certain experts who believe that many on-the-job accidents are caused by fatigue and sleep deprivation. I find this the argument unconvincing for several reasons. First and foremost, the author provides absolutely no evidence that overall worker productivity is attributable in part to the number of on-the-job accidents. Although common sense informs me that such a relationship exists, the author must provide some evidence of this cause-and-effect relationship before I can accept the author's final conclusion that the proposed course of action would in fact increase Alta's productivity. Secondly, the author assumes that some accidents at Alta are caused by fatigue or sleep deprivation. However, the author overlooks other possible causes, such as inadequate equipment maintenance or worker training, or the inherent hazards of Alta's manufacturing processes. By the same token, Panoply's comparatively low accident rate might be attributable not to the length of its work shifts but rather to other factors, such as superior equipment maintenance or worker training. In other words, without ruling out alternative causes of on-the-job accidents at both companies, the author cannot justifmbly conclude that merely by emulating Panoply's work-shift policy Alta would reduce the number of such accidents. Thirdly, even assuming that Alta's workers are fatigued or sleep-deprived, and that this is the cause of some of Alta's on-the-job accidents, in order to accept the author's solution to this problem we must assume that Alta's workers would use the additional hour of free time to sleep or rest. However, the author provides no evidence that they would use the time in this manner. It is entirely possible that Alta's workers would use that extra hour to engage in some other fatiguing activity. Without ruling out this possibility the author cannot convincingly conclude that reducing Alta's work shifts by one hour would reduce Alta's accident rate. Finally, a series of problems with the argument arise from the scant statistical information on which it relies. In comparing the number of accidents at Alta and Panoply, the author fails to consider that the per-worker accident rate might reveal that Alta is actually safer than Panoply, depending on the total number of workers at each company. Second, perhaps accident rates at the two companies last year were aberrations, and during other years Alta's accident rate was no greater, or even lower, than Panoply's rate. Or perhaps Panoply is not representative of industrial companies generally, and that other companies with shorter work shifts have even higher accident rates. In short, since the argument relies on very limited statistical information I cannot take the author's recommendation seriously. In conclusion, the recommendation for emulating Panoply's work-shift policy is not well supported. To convince me that shorter work shifts would reduce Alta's on-the-job accident rate, the author must provide clear evidence that work-shift length is responsible for some of Alta's accidents. The author must also supply evidence to support her final conclusion that a lower accident rate would in fact increase overall worker productivity.

The following appeared in a memo from the vice president of marketing at Dura-Sock, Inc. "A recent study of our customers suggests that our company is wasting the money it spends on its patented Endure manufacturing process, which ensures that our socks are strong enough to last for two years. We have always advertised our use of the Endure process, but the new study shows that despite our socks' durability, our average customer actually purchases new Dura-Socks every three months. Furthermore, our customers surveyed in our largest market, northeastern United States cities, say that they most value DuraSocks' stylish appearance and availability in many colors. These findings suggest that we can increase our profits by discontinuing use of the Endure manufacturing process." 1. Write a response in which you examine the stated and/or unstated assumptions of the argument. Be sure to explain how the argument depends on these assumptions and what the implications are for the argument if the assumptions prove unwarranted. Write a response in which you discuss what specific evidence is needed to evaluate the argument and explain how the evidence would weaken or strengthen the argument.

2.

GRE AWA John

3.

Write a response in which you discuss what questions would need to be answered in order to decide whether the recommendation and the argument on which it is based are reasonable. Be sure to explain how the answers to these questions would help to evaluate the recommendation.

Intro: The vice president of marketing at Dura-Sock, Inc. is offering a potentially harmful investment recommendation by claiming that Dura-Sock should discontinue its use of the Endure process. To support his recommendation, he points out a study that Dura-Sock customers actually purchase the socks every three months and a survey that reveals that Dura-Sock customers like the socks stylish appearance and availability in many colors. The study and survey, however, are insufficient in supporting his proposal, and the VP makes several unwarranted assumptions. Intro (simplified): The VP states that though Dura-Socks last for two years, customers buy the socks every three months. Therefore, he assumes that the consumers motive for buying the produce is not its durability.However, the author fails to rule out other possible motivation for consumption. Even if the survey is reliable, the author should consider the rest of the market. Vague terms: wasting moneyprecisely how much are they wasting? Study/survey error The company must calculate the outcome of such momentous decision. Study participants comment that they prefer Dura-Sock for its stylishness and availability might take DuraSocks enduring quality for granted.

" , 2 Endure . , . , . , ." : These findings suggest that Dura0Sock can increase its profits by discontinuing its use of the Endure manufacturing process. 1. survey ? survey.. 2. 3. . . 4. (retail) (whole) .

The following appeared in a business magazine. "As a result of numerous complaints of dizziness and nausea on the part of consumers of Promofoods tuna, the company requested that eight million cans of its tuna be returned for testing. Promofoods concluded that the canned tuna did not, after all, pose a health risk. This conclusion is based on tests performed on samples of the recalled cans by chemists from Promofoods; the chemists found that of the eight food chemicals most commonly blamed for causing symptoms of dizziness and nausea, five were not found in any of the tested cans. The chemists did find small amounts of the three remaining suspected chemicals but pointed out that these occur naturally in all canned foods." Write a response in which you discuss what questions would need to be addressed in order to decide whether the conclusion and the argument on which it is based are reasonable. Be sure to explain how the answers to the questions would help to evaluate the conclusion.

GRE AWA John

Representativeness of the tested cans. They should conduct a comparative study. The testing could be biased because Promofoods employees conducted the testing. How much (quantity) of the five and three suspected chemicals were in the canned foods. False cause: The substance that caused dizziness and nausea may not be one of the eight common chemicals.

" , Promofoods 8 . , . 8 5 . 3 ." : Promofoods concluded that the cans did not, after all, contain chemicals that posed a health risk 1. . 2. 8 . 3. 3 , . This magazine article concludes that the 8 million cans of tuna Promofoods recalled, due to complaints about nausea and dizziness, do not after ail contain any chemicals that pose a health risk. To support this conclusion the author cites the fact that five of eight chemicals commonly causing these symptoms were not found in the recalled cans, while the other three also occur naturally in other canned foods. For several reasons, this evidence lends little credible support to the author's conclusion. To begin with, the author relies partly on the fact that, although three of the eight chemicals most commonly blamed for nausea and dizziness appeared in Promofoods' recalled tuna, these chemicals also occur naturally in other canned foods. However, this fact alone lends no support to the author's conclusion, for two reasons. First, the author might be ignoring an important distinction between "naturally occurring" chemicals and those not occurring naturally. It is entirely possible that these three chemicals do not occur naturally in Promofoods' tuna, and that it is for this reason that the chemicals cause nausea and dizziness. Secondly, it is entirely possible that even when they occur naturally these chemicals cause the same symptoms. Unless the author rules out both possibilities, he cannot reliably conclude that the recalled tuna would not cause these symptoms. Another problem with the argument is that the author's conclusion is too broad. Based on evidence about certain chemicals that might cause two particular heath-related symptoms, the author concludes that the recalled tuna contains no chemicals that pose a health risk. However, the author fails to account for the myriad of other possible health risks that the recalled tuna might potentially pose. Without ruling out all other such risks, the author cannot justifiably reach his conclusion. A third problem with the argument involves that fact that the eight particular chemicals with which the test was concerned are only the eight "most commonly blamed" for nausea and dizziness. It is entirely possibly that other chemicals might also cause these symptoms, and that one or more of these other chemicals actually caused the symptoms. Without ruling out this possibility, the author cannot jusufiably conclude that the recalled tuna would not cause nausea and dizziness. A final problem with the argument involves thetesting procedure itself. The author provides no information about the number of recaUed cans tested or the selection method used. Unless the number of cans is a sufficiently large sample and is statistically repre sentative of all the recalled cans, the study's results are not statistically reliable. In conclusion, the article is unconvincing as it stands. To strengthen the assertion that the recalled tuna would not cause nausea and dizziness, the author must provide evidence that the three chemicals mentioned that occur naturally in other canned foods also appear naturally

GRE AWA John

in Promofoods' tuna. The author must also provide evidence that ingesting other canned foods containing these three chemicals does not cause these symptoms. To better evaluate the argument, we would need to know whether the sample used in the tests was statistically significant and representative of all the recalled tuna. We would also need to know what other chemicals in the recalled tuna might pose any health risk at all.

5:
Nature's Way, a chain of stores selling health food and other health-related products, is opening its next franchise in the town of Plainsville. The store should prove to be very successful: Nature's Way franchises tend to be most profitable in areas where residents lead healthy lives, and clearly Plainsville is such an area. Plainsville merchants report that sales of running shoes and exercise clothing are at all-time highs. The local health club has more members than ever, and the weight training and aerobics classes are always full. Finally, Plainsville's schoolchildren represent a new generation of potential customers: these schoolchildren are required to participate in a fitness-for-life program, which emphasizes the benefits of regular exercise at an early age. Write a response in which you examine the stated and/or unstated assumptions of the argument. Be sure to explain how the argument depends on these assumptions and what the implications are for the argument if the assumptions prove unwarranted.

False cause: First of all, the author believes that the Increased sales of running shoes and exercise clothing indicates Plainesville residents interest in leading healthy lives. However, this assumption is not logically convincing for several reasons. could be a fashion trend. Time shift: Fitness for life might not have any influence on schoolchildren as they grow. False cause: There could be other reasons for member increase in the health club. All of the above are insufficient condition. The author has to prove that local residents are interested in leading healthy lives. However, he supports his conclusion with insufficient evidence. Nevertheless, even if the residents are concerned with health, natures way may not be successful. First,

" , . Plainsville . . 5 , . . , "Fitness for Life" , . : We should therefore build our next new store in Plainsville.

GRE AWA John

: . 5 .( , ) tourist . . ( , .) .

Introduction Support In this memorandum, the author asserts that Natures Way should build its next new store in Plainsville. To support this assertion, the author states that Plainsville's merchants sales of exercise clothing are going well, the local health club has more members than ever, and a new generation of customers will help to ensure Natures Ways success. At first glance, the authors assumption seems convincing, but in-depth scrutiny reveals that it lacks substantial evidence as it stands. Body 1-Sampling Topic Sentence 1 To begin with, the author assumes that the merchants report indicates that the residents are concerned about their health. However, this assumption is based on unsubstantiated data. Example 1 (Rebuttal1) First, if we do not know the total volume of items sold and the price of the goods exactly, we cannot infer whether the residents are actually buying many goods. Example 2 (Rebuttal2) In addition to that, the report emphasizes the rising sales of running shoes and exercise clothing; however, these may not be hot-selling items for Natures Way or may not be the products the company is planning to sell. Concluding Sentence Therefore, in order to make the argument reliable, the author should reconsider the merchants report with more detailed data. Body 2-Causal Topic Sentence 2 Second, the author contends that the health club's classes are full, yet this does not mean that many people actually use the health club; other factors may be the real cause for those closed classes. Example 1 (Rebuttal1) To begin with, if the health club is very small, the number of people working out wouldnot be a large one. In fact, regular gym-going may just be a vogue among a small, unrepresentative segment of Plainsville's population. Example 2 (Rebuttal2) Moreover, it is possible that most of the people who exercise in the health club do weight training and aerobics only to look good and to meet other singles, not for their health. In that case, there would be little demand for health products. Concluding Sentence Thus, the author should not hasten to presume what really caused people to be interested in a healthier lifestyle and enroll in the health club. Body 3-Time-Shift Topic Sentence 3 Finally, the author highlights that Natures Way can expect a new generation of customers in Plainsville that will help the company in the long term. This notion is mistaken in that it assumes the conditions of the present will continue unchanged in the future. Although the school children are required to participate in the "fitness for life" program, they may not necessarily buy Natures Way's products. Example 1 (Rebuttal1) In the first instance, they may suffer a fall in purchasing power arising from future economic difficulties; this would cause reluctance to spend a considerable amount of money on health products, which tend to be more expensive. Example 2 (Rebuttal2) Another possibility is that there may emerge many competitor companies vying with Natures Way so that, in the future, the school children may not feel the necessity to purchase one companys health products over anothers.

GRE AWA John

Concluding Sentence Thus, the authors assumption is highly speculative since it relies heavily on unknowable future circumstances.
Conclusion Thesis In sum, the author uses many assumptions that are insufficient in supporting his claims. Support In order for the authors claims to be convincing, he needs to advance more persuasive evidence that people in Plainsville really are concerned with their health and health food.
The following was written as a part of an application for a small-business loan by a group of developers in the city of Monroe. "A jazz music club in Monroe would be a tremendously profitable enterprise. Currently, the nearest jazz club is 65 miles away; thus, the proposed new jazz club in Monroe, the C-Note, would have the local market all to itself. Plus, jazz is extremely popular in Monroe: over 100,000 people attended Monroe's annual jazz festival last summer; several well-known jazz musicians live in Monroe; and the highest-rated radio program in Monroe is 'Jazz Nightly,' which airs every weeknight at 7 P.M. Finally, a nationwide study indicates that the typical jazz fan spends close to $1,000 per year on jazz entertainment." 1. 2. Write a response in which you discuss what specific evidence is needed to evaluate the argument and explain how the evidence would weaken or strengthen the argument. Write a response in which you examine the stated and/or unstated assumptions of the argument. Be sure to explain how the argument depends on these assumptions and what the implications are for the argument if the assumptions prove unwarranted. Write a response in which you discuss what questions would need to be answered in order to decide whether the prediction and the argument on which it is based are reasonable. Be sure to explain how the answers to these questions would help to evaluate the prediction.

3.

Group error: nationwide survey may not reflect local trends. Is the nationwide jazz fan population substantial. Insufficient: non-residents of Monroe may have attended the jazz festival. (Body alternative explanation, last year may have been an anomaly. The author should consider data from various years.) The author should indicate how many out of 100,000 were Monroe residents. Nationwide study: Does this reflect Insufficient: Citizens of Monroe may continue to go to the jazz club 65 miles away. Are the people in Monroe really interested in jazz? Majority of the people who attended the jazz festival might not be Monroe residents. Survey error: nationwide study may not be applicable to Monroe Jazz musicians who live in Monroe.. Monopoly Radio station: In this business application, the author claims that the proposed jazz club, C Note, will be very profitable in Monroe. To support this claim, the author argues for his case with several evidences. At first glance the authors argument seems convincing; however, careful scrutiny reveals that his argument in specious. To begin with, the author claims that Monroes citizens are interested in jazz. He presents three evidences: First,.. Second, Third, However, "Monroe . , 65 . , C Note . , . 10 Morone , , Jazz Nightly . 1 . C Note ." : . It is clear that the C Note cannot help but make money

GRE AWA John

: nearest jazz club . Festival jazz . jazz . . () . _Ms. Noh #6 In this application, the author suggests that a jazz club in Monroe will make a number of profits. To support this suggestion, the author exemplifies the local condition, popularity of jazz in Monroe, and nationwide study. However, careful scrutiny of each of the facts reveals that it provides little credible support for the authors recommendation. Good clear intro. First, the author assumes that jazz is popular in Monroe because of several facts: the jazz festival last year had high participation, some famous jazz musicians live in Monroe, and the high-rated radio program is Jazz Nightly. However, this assumption has many drawbacks that must be seriously considered(Good topic sentences). If many attendants in the last-years festival came from other cities and not Monroe, it is hard to conclude that Monroes people like jazz. Therefore, the author must examine how many Monroe residents actually attended the festival. On top of that, there is little relationship between habitation of famous jazz musician and the popularity of jazz in Monroe. Although several well-known musicians live there, if they do not take part in any jazz performance of Monroe, this might have no effect to the interest of Monroes residents about jazz. Finally, in the case of radio program, this is also not suitable reason why jazz is popular in Monroe. It might be possible that people cannot help choosing Jazz Nightly because there are few radio programs at Night. The fact that the radio program is the highest rating program is not a germane evidence. The approximate number of listeners would be the more crucial evidence. Therefore, the author needs to seriously deliberate the correlation between jazzs popularity in Monroe and his examples. (Good logical flow and clarity) Second, the author uses as evidence the nationwide study that jazz fans spend much money on jazz entertainment to substantiate why starting a jazz club in Monroe will be profitable. In other words, the author assumes that the characteristics of a nationwide study can be applied to Monroe. The national study would lend support to the applicants claim only if residents in Monroe typify national jazz fans. However, the author does not provide credible evidence that this is the case. Moreover, the populations of jazz fans nationwide may be insubstantial. Thus, the author should not infer hastily that Monroes residents will spend much money on enjoying jazz from the nationwide study. Lastly, even if jazz is popular in Monroe, C Note may not be successful. It is entirely possible that residents might still prefer other clubs where they have always went. In addition, there is another possibility that the nearest jazz club will attract many of Monroes people because it serves fine performances and is equipped with favorite facilities. Without considering these other possibilities, the author cannot make his argument convincing. In sum, the author presents many reasons that are insufficient in supporting his or her claim. In order for the authors claims to be convincing, he needs to advance more persuasive evidence such as the total number of Monroe residents who attended the jazz festival, the effects on the popularity of jazz by the musicians living in Monroe, and the actual number of residents who would typify themselves to be jazz fans through a local survey. Without substantial evidence that C Note will be successful in Monroe, the businessmen may be overinvesting in what might lead to a business failure. . Excellent clarity. Score: 5.0

GRE AWA John

The following appeared in a newsletter offering advice to investors. "Over 80 percent of the respondents to a recent survey indicated a desire to reduce their intake of foods containing fats and cholesterol, and today low-fat products abound in many food stores. Since many of the food products currently marketed by Old Dairy Industries are high in fat and cholesterol, the company's sales are likely to diminish greatly and company profits will no doubt decrease. We therefore advise Old Dairy stockholders to sell their shares, and other investors not to purchase stock in this company." Write a response in which you discuss what questions would need to be answered in order to decide whether the advice and the argument on which it is based are reasonable. Be sure to explain how the answers to these questions would help to evaluate the advice.

Survey 80 Old Dairy could change their products and manufacture low fat dairy foods. Less competing companies. Old Dairy could eventually be the only company that produces . Imprecise numbers and measurements. Customers may still buy high fat dairy products. The author of the newsletter is offering potentially dangerous advice by recommending Old Dairy stockholders to withdraw investment and stop purchase. What is more, the authors prediction debases the reputation and business of Old Dairy and if false could devoid the investment opportunity of the newsletter readers. Therefore, investors should examine whether the authors evidences are substantial. To begin with, the author states that 80 percent of the respondents in a survey indicated a desire to reduce their intake of foods. He therefore argues that Old Dairys high fat and cholesterol products would decrease in sales. However, the author makes a crucial error in this argument. First, the author provides no evidence that the surveys results are statistically reliable. Were they representative of all the customers? Were they chosen for the survey randomly? Furthermore, the desire to reduce fat and cholesterol intake is a pervasive trend in todays opulent society; however, the author erroneously identifies this as a new phenomenon which will affect consumer trends. Second, having a desire to reduce fat and cholesterol intake does not necessarily indicate that people who have this desire will actually reduce consuming these types of products. It is entirely possible that they may continue buying Old Dairy products for its quality and taste. Accordingly, the author cannot draw any firm conclusion that people will not buy Old Dairy products. Therefore, if any of these cases are true, the author may be offering investors a detrimental investment advice. " 80% . . Old Dairy Industries , , . , , ." : Old Dairy stockholders to sell their shares and other investors not to purchase stock in this company : ( ). . .( . ) .
The following appeared in a letter to the editor of the Balmer Island Gazette. "On Balmer Island, where mopeds serve as a popular form of transportation, the population increases to

GRE AWA John

100,000 during the summer months. To reduce the number of accidents involving mopeds and pedestrians, the town council of Balmer Island should limit the number of mopeds rented by the island's moped rental companies from 50 per day to 25 per day during the summer season. By limiting the number of rentals, the town council will attain the 50 percent annual reduction in moped accidents that was achieved last year on the neighboring island of Seaville, when Seaville's town council enforced similar limits on moped rentals." 1. Write a response in which you discuss what questions would need to be answered in order to decide whether the recommendation is likely to have the predicted result. Be sure to explain how the answers to these questions would help to evaluate the recommendation. Write a response in which you discuss what questions would need to be answered in order to decide whether the prediction and the argument on which it is based are reasonable. Be sure to explain how the answers to these questions would help to evaluate the prediction. Write a response in which you examine the stated and/or unstated assumptions of the argument. Be sure to explain how the argument depends on these assumptions and what the implications are for the argument if the assumptions prove unwarranted.

2.

3.

Whats the actual population of Balmer Island? 100,000is this a significant increase? What kind of accidents? Skin abrasions or serious injury? And compared to Seaville, how serious are the accidents and the actual number of accidents? Did Seaville enforce other restrictions like safety signs?How different are the conditions of Balmer and Seaville regarding population, road (safety) conditions, topography, other town-government regulation? How much will the economy of Balmer be affected do to this restriction? Could it cause an economic recession due to the fact that these rental companies chance to make money is only during the summer, thereby weakening the economic infrastructure? Are there any other ways that could better alleviate the accident rate?

Statistics: 50%-imprecise Analogy: Balmer compared with Torseau False Cause: Accidents might have occurred because of reasons other than mopeds, False Cause: population increase may not be part of the cause of the accidents Other explanations for the accident: pedestrians, few road safety regulations, narrow roads There could be other better solutions "Balmer Island . 2 , 6 2 50 30 . , Torseau 50% 50% ." : The town council of Balmer Island should linit the number : . ( .) .( .) , .
In this letter, the author recommends that Balmer Island should limit the number moped rentals from 50 to 30 per day. To support this recommendation, the author points out several reasons. However, careful scrutiny of each of the facts reveals that it is filled with unanswered questions that could significantly weaken the authors recommendation / with loops and holes which are answered. The recommendation depends on the assumption that no alternative means of reducing the number of accidents are available. However, the author fails to offer any evidence to substantiate this crucial assumption. It is highly possible that means other than this would better solve the problem. Perhaps they could widen the roads or put-up more safety signs. Or perhaps the accidents were due to the lack of skills in which case proper safety training would significantly alleviate the problem. Without considering and ruling out these and other alternative means of reducing accidetns, the author cannot confidently conclude that merely emulating Torseau would suffice. Moreover, the author is advising a recommendation which could potentially harm the economy of Balmer Island since Moreover, the Balmer Island should alternative means to reduce accidents because limiting moped rentals during the summer could harm the economy of Balmer First of all, the author believes that increase in population and the number of moped rentals are responsible for the accidents. It is entirely possible that other factors are responsible for the accidents. Perhaps, Balmer Islands lack of safety signs was a major factor. Or maybe the roads are narrow and dangerous on the Island; therefore, the town council could

GRE AWA John

enforce stricter traffic regulations to alleviate the problem. Accordingly, if either of these scenarios is true, the author cannot draw any firm conclusion that increase in the number of population and moped rentals are the cause of the accidents. The author of this editorial recommends that to reduce accidents involving mopeds and pedestrians Balmer Island's city council should restrict moped rentals to 30 per day, down from 50, at each of the island's six rental outlets. To support this recommendation the author cites the fact that last year, when nearby Torseau Island's town council enforced similar measures, Torseau's rate of moped accidents fell by 50%. For several reasons, this evidence provides scant support for the author's recommendation. To begin with, the author assumes that all other conditions in Balmer that might affect the rate of moped-pedestrian accidents will remain unchanged after the restrictions are enacted. However, with a restricted supply of rental mopeds people in Balmer might purchase mopeds instead. Also, the number of pedestrians might increase in the future; with more pedestrians, especially tourists, the risk of moped-pedestrian accidents would probably increase. For that matter, the number of rental outlets might increase to make up for the artificial supply restriction per outlet--a likely scenario assuming moped rental demand does not decline. Without considering and ruling out these and other possible changes that might contribute to a high incidence of moped-pedestrian accidents, the author cannot convince me that the proposed restrictions will necessarily have the desired effect. Next, the author fails to consider other possible explanations for the 50% decline in Torseau's moped accident rate last year. Perhaps last year Torseau experienced unusually fair weather, during which moped accidents are less likely. Perhaps fewer tourists visited Tot sean last year than during most years, thereby diminishing the demand for rental mopeds to below the allowed limits. Perhaps last year some of Torseau's moped rental outlets purchased new mopeds that are safer to drive. Or perhaps the restrictions were already in effect but were not enforced until last year. In any event, a decline in Torseau's moped accident rate during only one year is scarcely sufficient to draw any reliable conclusions about what might have caused the decline, or about what the accident rate will be in years ahead. Additionally, in asserting that the same phenomenon that caused a 50% decline in moped accidents in Torseau would cause a similar decline in Balmer, the author relies on what might amount to an unfair analogy between Balmer and Torseau. Perhaps Balmer's ability to enforce moped-rental restrictions does not meet Torseau's ability; if not, then the mere enactment of similar restrictions in Balmer is no guarantee of a similar result. Or perhaps the demand for mopeds in Torseau is always greater than in Balmer. Specifically, if fewer than all available mopeds are currently rented per day from the average Balmer outlet, while in Torseau every available moped is rented each day, then the proposed restriction is likely to have less impact on the accident rate in Balmer than in Torseau. Finally, the author provides no evidence that the same restrictions that served to reduce the incidence of all "moped accidents" by 50% would also serve to reduce the incidence of accidents involving "mopeds and pedestrians" by 50%. Lacking such evidence, it is entirely possible that the number of moped accidents not involving pedestrians decreased by a greater percentage, while the number of moped-pedestrian accidents decreased by a smaller percentage, or even increased. Since the author has not accounted for these possibilities, the editorial's recommendation cannot be taken seriously. In conclusion, the recommendation is not well supported. To convince me that the proposed restriction would achieve the desired outcome, the author would have to assure me that no changes serving to increase Balmer's moped-pedestrian accident rate will occur in the foreseeable future. The author must also provide dear evidence that last year's decline in moped accidents in Torseau was attributable primarily to its moped rental restrictions rather than to one or more other factors. In order to better evaluate the recommendation, I would need more information comparing the supply of and demand for moped rentals on the two islands. I would also need to know the rate of mopedpedestrian accidents in Torseau both prior to and after the restrictions were enforced in Torseau.

The following appeared in a magazine article about planning for retirement. "Clearview should be a top choice for anyone seeking a place to retire, because it has spectacular natural beauty and a consistent climate. Another advantage is that housing costs in Clearview have fallen significantly during the past year, and taxes remain lower than those in neighboring towns. Moreover, Clearview's mayor promises many new programs to improve schools, streets, and public services. And best of all, retirees in Clearview can also expect excellent health care as they grow older, since the number of physicians in the area is far greater than the national average." Write a response in which you discuss what specific evidence is needed to evaluate the argument and explain how the evidence would weaken or strengthen the argument.

GRE AWA John

-Natural beauty and consistent climate may not be the most wanted qualities. -Housing costs could have lowered on a national level; wealthy retirees may not care about costs -Taxes may be high compared to the nations average tax rate. -What about other qualities of Clearview? Crime rate; what qualities would retirees want? -If schools, streets, and public services need improvement, then this is proof that the current condition of Clearview is low. Or due to budgetary reasons, the mayor may not follow-up on his promise because of lowered tax rate. -Schools and people who are retired: no relationship. -Physicians: What kind of physicians? Number is irrelevant. Are these physicians capable of addressing the illnesses of old people? This author argues that anyone seeking a place to retire should choose Clearview. To support this argument the article cites Clearview's consistent climate and natural beauty; it's falling housing costs; its low property taxes compared to nearby towns; and the mayor's promise to improve schools, streets, and services. The article also claims that retirees can expect excellent health care because the number of physicians in Clearview greatly exceeds the national average. This argument is flawed in several critical respects. To begin with, although consistent climate and natural beauty might be attractive to many retirees, these features are probably not important to all retirees. For many retirees it is probably more important to live near relatives, or even to enjoy changing seasons. Thus I cannot accept the author's sweeping recommendation for all retirees on this basis. Also, Clearview's declining housing costs do not necessarily make Clearview the best place to retire for two reasons. First, despite the decline Clearview's housing costs might be high compared to housing costs in other cities. Secondly, for wealthier retirees housing costs are not likely to be a factor in choosing a place to retire. Thus the mere fact that housing costs have been in decline lends scant support to the recommendation. The article's reliance on Clearview's property-tax rates is also problematic in two respects. First, retirees obviously have innumerable choices about where to retire besides Clear view and nearby towns. Secondly, for retirees who are well-off financially property taxes are not likely to be an important concern in choosing a place to retire. Thus it is unfair to infer from Clearview's property-tax rates that retirees would prefer Clearview. Yet another problem with the argument involves the mayor's promises. In light of Clearview's low property-tax rates, whether the mayor can follow through on those promises is highly questionable. Absent any explanation of how the city can spend more money in the areas cited without raising property taxes, I simply cannot accept the editorial's recommendation on the basis of those promises. Besides, even if the city makes the improvements promised, those improvements--particular the ones to schools--would not necessarily be important to retirees. Finally, although the number of physicians in Clearview is relatively high, the per capita number might be relatively low. Moreover, it would be fairer to compare this per capita number with the per capita number for other attractive retirement towns--rather than the national average. After all, retirees are likely to place a relatively heavy burden on health-care resources. Besides, the article provides no assurances that the number of physicians in Clearview will remain high in the foreseeable future. In conclusion, the recommendation is poorly supported. To strengthen it the author must convince me--perhaps by way of a reliable survey--that the key features that the vast majority of retirees look for in choosing a place to live are consistent climate, natural beauty, and low housing costs. The author must also provide better evidence that Clear view's property taxes are lower than the those of cities in other areas. The author must also explain how the city can make its promised improvements without raising property taxes. Finally, to better assess the argument I would need to now how the per capita number of physicians in Clearview would compare to the national average in the future.

The following appeared as a letter to the editor from a Central Plaza store owner. "Over the past two years, the number of shoppers in Central Plaza has been steadily decreasing while the popularity of skateboarding has increased dramatically. Many Central Plaza store owners believe that the decrease in their business is due to the number of skateboard users in the plaza. There has also been a dramatic increase in the amount of litter and vandalism throughout the plaza. Thus, we recommend that the city prohibit skateboarding in Central Plaza. If skateboarding is prohibited here, we predict that business in Central Plaza will return to its previously high levels." Write a response in which you discuss what questions would need to be answered in order to decide whether the recommendation is likely to have the predicted result. Be sure to explain how the answers to these questions would help to evaluate the recommendation.
Why two years ago? What happened two years ago which started this decline? Is the dramatic increase in the popularity of skateboarding the cause of the steady decline of shoppers? Are there any malls nearby? Were there any changes nearby which could affect the decline in customersa big mall perhaps? Could the decline be due to the shop owners?

GRE AWA John

How many skateboarders use the plaza? Where do they skateboard? Do they shop and are they customers? Are the increase in litter and vandalism due to skateboarders? Could this be alleviated by installing CCTVs and hiring security?

This editorial concludes that the city should ban skateboarding from its downtown Central Plaza in order to attract visitors to that area, to return the area to its "former glory," and to make it "a place where people can congregate for fun and relaxation." To justify this conclusion the editorial points out that skateboarders are nearly the only people one sees anymore at Central Plaza, and that the Plaza is littered and its property defaced. The editorial also points out that the majority of downtown merchants support the skate boarding ban. This argument is flawed in several critical respects. First, the editorial's author falsely assumes that a ban on skateboarding is both necessary and sufficient to achieve the three stated objectives. Perhaps the city can achieve those objectives by other means as well--for example, by creating a new mall that incorporates an attractive new skateboard park. Even if banning skateboarders altogether is necessary to meet the city's goals, the author has not shown that this action by itself would suffice. Assuming that the Plaza's reputation is now tarnished, restoring that reputation and, in turn, enticing people back to the Plaza might require additional measures--such as removing litter and graffiti, promoting the Plaza to the public, or enticing popular restaurant or retail chains to the Plaza. Secondly, the editorial assumes too hastily that the Plaza's decline is attributable to the skateboarders--rather than to some other phenomenon. Perhaps the Plaza's primary appeal in its glory days had to do with particular shops or eateries, which were eventually replaced by less appealing ones. Or perhaps the crime rate in surrounding areas has risen dramatically, for reasons unrelated to the skateboarders' presence at the Plaza. Without ruling out these and other alternative explanations for the Plaza's decline, the editorial's author cannot convince me that a skateboard ban would reverse that decline. Thirdly, the editorial's author might be confusing cause with effect--by assuming that the skateboarders caused the abandonment of the Plaza, rather than vice versa. It is entirely possible that skateboarders did not frequent the Plaza until it was largely abandoned--and because it had been abandoned. In fact this scenario makes good sense, since skateboarding is most enjoyable where there are few pedestrians or motorists to get in the way. Fourth, it is unreasonable to infer from the mere fact that most merchants favor the ban that the ban would be effective in achieving the city's objectives. Admittedly, perhaps these merchants would be more likely to help dean up the Plaza area and promote their businesses were the city to act in accordance with their preference. Yet lacking any supporting evidence the author cannot convince me of this. Thus the survey amounts to scant evidence at best that the proposed ban would carry the intended result. Finally, the author recommends a course of action that might actually defeat the city's objective of providing a fun and relaxing place for people to congregate. In my experience skateboarding contributes to an atmosphere of fun and relaxation, for adults and children alike, more so than many other types of ambiance. Without considering that continuing to allow skateboarding--or even encouraging this activity--might achieve the city's goal more effectively than banning the activity, the author cannot convincingly conclude that the ban would be in the city's best interests. In sum, the argument is a specious one. To strengthen it, the editorial's author must provide dear evidence that skateboarding, and not some other factor, is responsible for the conditions marking the Plaza's decline. The author must also convince me that no alternative means of restoring the Plaza are available to the city, and that the proposed ban by itself would suffice to attract tourists and restore the Plaza to its former glory. Finally, to better assess the argument it would be useful to know the circumstances under which the downtown merchants would be willing to help the city achieve its objectives.

GRE AWA John

6:
The following recommendation appeared in a memo from the mayor of the town of Hopewell. "Two years ago, the nearby town of Ocean View built a new municipal golf course and resort hotel. During the past two years, tourism in Ocean View has increased, new businesses have opened there, and Ocean View's tax revenues have risen by 30 percent. Therefore, the best way to improve Hopewell's economyand generate additional tax revenuesis to build a golf course and resort hotel similar to those in Ocean View." Write a response in which you examine the stated and/or unstated assumptions of the argument. Be sure to explain how the argument depends on these assumptions and what the implications are for the argument if the assumptions prove unwarranted.

Assumptions: The author assumes that OVs municipal golf course and resort hotel caused tourism, new businesses and increased tax revenues. There may be other reasons: advertising, promo, He assumes that this will continue. Assumes that Ocean View and Hopewell are similar in many waysthe name suggests otherwise. OV may have always been a tourist attractions for its beaches. We need to know the topography.

"2, Ocean View . 2 , . 30% . Hopewell , Ocean View ." 1. . , . 2. . 3. 2 . .


In this memo HopeweU's mayor recommends that in order to stimulate the town's economy and boost tax revenues HopeweU should build a new golf course and resort hotel, just as the town of Ocean View did two years ago. To support this recommendation the mayor points out that in Ocean View during the last two years tourism has increased, new businesses have opened, and tax revenues have increased by 30%. I find the mayor's argument unconvincing in several important respects. First of all, it is possible that the mayor has confused cause with effect respecting the recent developments in Ocean View. Perhaps Ocean View's construction of a new golf course and hotel was a response to previous increases in tourism and business development increases that have simply continued during the most recent two years. Since the mayor has failed to account for this possibility, the claim that Hopewell would boost its economy by also constructing a golf course and hotel is completely unwarranted. Secondly, the mayor fails to account for other possible causes of the trends in Ocean View during the last two years. The increase in tourism might have been due to improving economic conditions nationwide, or to unusually pleasant weather in the region. The new businesses that have opened in Ocean View might have opened there irrespective of the new golf course and hotel. And, the 30% increase in tax revenues might have been the result of an increase in tax rates, or the addition of a new type of municipal tax. Without ruling out these and other alternative explanations for the three recent trends in Ocean View, the mayor cannot reasonably infer based on those trends that Hopewell's economy would benefit by following Ocean View's example. Thirdly, even if the recent trends in Ocean View are attributable to the construction of the new golf course and hotel there, the mayor assumes too hastily that the golf course and hotel will continue to benefit that town's overall economy. The mayor has not accounted for the possibility that increased tourism will begin to drive residents away during tourist season, or

GRE AWA John

that new business development will result in the town's losing its appeal as a place to visit or to live. Unless the mayor can convince me that these scenarios are unlikely I cannot accept the mayor's recommendation that Hopewell follow Ocean View's example. Finally, the mayor's argument rests on the unsubstantiated assumption that Hopewell and Ocean View are sufficiently alike in ways that might affect the economic impact of a new golf course and hotel. Hopewell might lack the sort of natural environment that would attract more tourists and new businesses to the town--regardless of its new golf course and hotel. For that matter, perhaps Hopewell already contains several resort hotels and golf courses that are not utilized to their capacity. If so, building yet another golf course and hotel might amount to a misallocation of the town's resources--and actually harm the town's overall economy. In sum, the mayor's recommendation is not well supported. To bolster it the mayor must provide better evidence that Ocean View's new golf course and hotel and not some other phenomenon--has been responsible for boosting Ocean View's economy during the last two years. To better assess the recommendation I would need to know why Ocean View decided to construct its new golf course and hotel in the first place--specifically, what events prior to construction might have prompted that decision. I would also need to thoroughly compare HopeweU with Ocean View--especially in terms of their appeal to tourists and businesses--to determine whether the same course of action that appears to have boosted Ocean View's economy would also boost Hopewell's economy.

The following is part of a memorandum from the president of Humana University. "Last year the number of students who enrolled in online degree programs offered by nearby Omni University increased by 50 percent. During the same year, Omni showed a significant decrease from prior years in expenditures for dormitory and classroom space, most likely because instruction in the online programs takes place via the Internet. In contrast, over the past three years, enrollment at Humana University has failed to grow, and the cost of maintaining buildings has increased along with our budget deficit. To address these problems, Humana University will begin immediately to create and actively promote online degree programs like those at Omni. We predict that instituting these online degree programs will help Humana both increase its total enrollment and solve its budget problems." Write a response in which you discuss what questions would need to be answered in order to decide whether the prediction and the argument on which it is based are reasonable. Be sure to explain how the answers to these questions would help to evaluate the prediction.

Is Omni University successful due to the online degree program? 50%? Is the decrease in expenditures for dormitory and classroom space due to the decrease in # of on-campus students? Which classes were successful? Does HU have those classes? Even if the long-distance degree programs at Omni University benefited the school, the presidents recommendation that Human College should emulate Omni University is too hasty. First, OUs name implies that

GRE AWA John

the school would have more majors than Humana the president should examine which degrees were in the long-distance program

" Omni 50% . , Omni , . , 3 , Humana . , Humana , Omni ." : we should initiate and actively promote long-distance degree programs like those at Omni." : . (causal, , ) bad analogy(omni university . )-> omni college . Humana Humana .( )

The following appeared as part of a business plan developed by the manager of the Rialto Movie Theater. "Despite its downtown location, the Rialto Movie Theater, a local institution for five decades, must make big changes or close its doors forever. It should follow the example of the new Apex Theater in the mall outside of town. When the Apex opened last year, it featured a video arcade, plush carpeting and seats, and a state-of-the-art sound system. Furthermore, in a recent survey, over 85 percent of respondents reported that the high price of newly released movies prevents them from going to the movies more than five times per year. Thus, if the Rialto intends to hold on to its share of a decreasing pool of moviegoers, it must offer the same features as Apex." Write a response in which you discuss what questions would need to be answered in order to decide whether the recommendation is likely to have the predicted result. Be sure to explain how the answers to these questions would help to evaluate the recommendation.

Before following through this business plan, the manager should investigate the cause of Rialtos unsuccessful business. The author provides no evidence that the surveys results are statistically reliable. The surveys sample of 85 percent must be sufficient in size and representative of overall population of the city where Rialto and Apex is serving. Lacking evidence of a sufficiently representative sample, the author cannot justifiably rely on the survey to draw any conclusion whatsoever. The author does not indicate that Apex is indeed currently successful. However, even if Apex is enjoying success, the argument relies on what might be a false analogy between Rialto and Apex. In order for Apex to serve as a model that Rialto should emulate, the author must assume that all relevant circumstances are essentially the same. However, this assumption is unwarranted. For example, the argument overlooks the face that Apex is located in a strategic placebeside a mall where customers can, not only watch a movie, but also enjoy shopping. Therefore, simply changing the facility to that of Apex may not lead to success.

GRE AWA John

The author does not mention whether Apex is successful or not. Nevertheless, even if Apex is currently successful, the argument relies on what might be a false analogy between Rialto and Apex. In order for Apex to serve as a model that Rialto should emulate, the author must assume that all relevant circumstances are essentially the same. However, this assumption is unwarranted. For example, the argument overlooks the fact that these two institutions are located in different locations: Rialto in downtown and Apex in a mall outside of town. Although Apex opened with state-of-the-art facilities, the decisive factor in its success could be due to its strategic location of being in a mall. People could enjoy both shopping and movies at one location, thus, they may prefer Apex over Rialto. Furthermore, the place where people enjoy leisure activities has shifted in the past decades for most cities from downtown to the suburbs. Therefore, Rialto may not be successful even if it emulates Apexs facilities. A better business plan may be relocating Apex to the thriving section of the downtown. "Rialto 50 , . Apex . Apex , , , . , , 85% 5 . , Rialto , Apex ." : , Apex . 1. . 2. apex . 3. . 4. rialto .( )

The following is a recommendation from the business manager of Monarch Books. "Since its opening in Collegeville twenty years ago, Monarch Books has developed a large customer base due to its reader-friendly atmosphere and wide selection of books on all subjects. Last month, Book and Bean, a combination bookstore and coffee shop, announced its intention to open a Collegeville store. Monarch Books should open its own in-store caf in the space currently devoted to children's books. Given recent national census data indicating a significant decline in the percentage of the population under age ten, sales of children's books are likely to decline. By replacing its children's books section with a caf, Monarch Books can increase profits and ward off competition from Book and Bean." Write a response in which you examine the stated and/or unstated assumptions of the argument. Be sure to explain how the argument depends on these assumptions and what the implications are for the argument if the assumptions prove unwarranted.

The following is a recommendation from the business manager of Monarch Books. "Since its opening in Collegeville twenty years ago, Monarch Books has developed a large customer base due to its reader-friendly atmosphere and wide selection of books on all subjects. Last month, Book and Bean, a combination bookstore and coffee shop, announced its intention to open a Collegeville store. Monarch Books should open its own in-store caf in the space currently devoted to children's books. Given recent national census data indicating a significant decline in the percentage of the population under age ten, sales of children's books are likely to decline. By replacing its children's books section with a caf, Monarch Books can increase profits and ward off competition from Book and Bean." 1. Write a response in which you discuss what questions would need to be answered in order to decide whether the recommendation is likely to have the predicted result. Be sure to explain how the answers to these questions would help to evaluate the recommendation. Write a response in which you discuss what specific evidence is needed to evaluate the argument

2.

GRE AWA John

and explain how the evidence would weaken or strengthen the argument.

No evidence regarding Monarch Books success. Even if Regal Books is successful, this may not be attributable to the caf. False analogy: Emulating may not lead to success. Other factors may be involved Insufficient condition: The national census is not enough evidence that childrens book sales will decline. Can the national census represent the local child population? Did opening a caf boost sales for Regal Books? Even assuming Regal is successful by opening a caf, this may not be suitable for Monarch, which plans to close the childrens book section to establish a cafe. Imprecise language: relatively little space how small? The managers recommendation contradicts what he says: Since Monarch is popular for its wide selection of books, closing a selection which targets a major group of readers may hurt Monarchs sales. Is this the best way to compete?

When Stanley Park first opened, it was the largest, most heavily used public park in town. It is still the largest park, but it is no longer heavily used. Video cameras mounted in the park's parking lots last month revealed the park's drop in popularity: the recordings showed an average of only 50 cars per day. In contrast, tiny Carlton Park in the heart of the business district is visited by more than 150 people on a typical weekday. An obvious difference is that Carlton Park, unlike Stanley Park, provides ample seating. Thus, if Stanley Park is ever to be as popular with our citizens as Carlton Park, the town will obviously need to provide more benches, thereby converting some of the unused open areas into spaces suitable for socializing. Write a response in which you examine the stated and/or unstated assumptions of the argument. Be sure to explain how the argument depends on these assumptions and what the implications are for the argument if the assumptions prove unwarranted.

Stanley , . , . , drop( ) . 50 . , Carlton 150 . Stanley , Carlton . , Stanley Carlton , , . ===>drop (.)---- a place or central depository to which something (as mail, money, or stolen property) is brought for distribution or transmission; also : the act of depositing something at such a place drop> : if Stanley Park is ever to be as popular with our citizens as is Carlton Park, the town will obviously need to provide more benches, thereby converting some of the unused open areas into spaces suitable for socializing. 1. ? . . 2. .( .) 3. . 4. . 5. .( , .) 6. . ( .)

GRE AWA John

Das könnte Ihnen auch gefallen